In a recent BMO survey, many Canadian university students said they expected to owe


$26, 500 after graduation. A group of n = 64 university students are randomly selected, and


the average student loan debt is found to be $26,000 with a standard deviation of 500.


(a) Construct a 98% confidence interval for the true average student loan debt for


university students in Canada.




(b) Does this contradict the reported average of $26,500? Explain

Answers

Answer 1

(a) With 98% confidence, the true average student loan debt for university students in Canada falls between $25,883.5 and $26,116.5.

(b) No, this does not contradict the reported average of $26,500.

(a) To construct a 98% confidence interval, we can use the formula:

CI = [tex]\bar{x}[/tex] ± z* (σ/√n)

where [tex]\bar{x}[/tex] is the sample mean, σ is the population standard deviation (unknown), n is the sample size, and z* is the z-value from the standard normal distribution that corresponds to the desired level of confidence (98% in this case).

Substituting the given values, we get:

CI = 26,000 ± 2.33 * (500/√64)
CI = 26,000 ± 116.5
CI = (25,883.5, 26,116.5)

Therefore, we can say with 98% confidence that the true average student loan debt for university students in Canada falls between $25,883.5 and $26,116.5.

(b) No, this does not contradict the reported average of $26,500. The reported average is within the confidence interval we calculated, which means that it is a plausible value for the true average. The sample mean of $26,000 is slightly lower than the reported average, but this could be due to random sampling error.

Overall, we cannot make a definitive conclusion about the true average based on this sample alone, but we can say with 98% confidence that it falls within the range of $25,883.5 to $26,116.5.

To know more about 98% confidence interval, refer to the link below:

https://brainly.com/question/30637686#

#SPJ11


Related Questions

Pedro is walking down the longest staircase ever, which contains 4000 steps. She


starts from the top and is walking down the staircase at 150 steps a minute. Hal is


walking up the staircase, starting at the bottom, at 80 steps a minute. After how


many minutes will they meet?

Answers

Pedro and Hal will walk 1800 steps staircase and meet after 25 minutes.

Pedro is walking down the longest staircase ever, which contains 4000 steps. She starts from the top and is walking down the staircase at 150 steps a minute. Hal is walking up the staircase, starting at the bottom, at 80 steps a minute. After how many minutes will they meet?

Let's assume that they will meet at point X, which is y steps away from the top and z steps away from the bottom. As Pedro is walking down the staircase, she will cover a distance of y steps, while Hal is walking up the staircase, he will cover a distance of (4000-z) steps.

The time taken by Pedro to cover a distance of y steps is y/150 minutes, while the time taken by Hal to cover a distance of (4000-z) steps is (4000-z)/80 minutes. Since they will meet at the same point X, we can set these two times equal to each other and solve for y and z.

y/150 = (4000-z)/80

Solving this equation, we get y = 1800 and z = 2200. This means that Pedro will have covered 1800 steps in y/150 = 12 minutes and Hal will have covered (4000-2200) = 1800 steps in (4000-2200)/80 = 25 minutes.

Know more about steps here:

https://brainly.com/question/1229805

#SPJ11

Which expressions represent the length of side MN?



Choose 2 answers:



A. 4 • sin (65)



B. 1. 9 • cos (65)



C. 4/sin (65)



D. 1. 9/cos (65)



PLEASE HELP!!

Answers

Expressions A and B represent the length of side MN.

How can the length of side MN be expressed?

The two expressions that represent the length of side MN are A) 4•sin(65) and B) 1.9•cos(65). The length of side MN can be found using the trigonometric ratios of sine and cosine in a right triangle.

The angle of 65 degrees is opposite to side MN and the hypotenuse of the triangle is given as 4 units. Using the sine ratio, we can find the length of MN as 4•sin(65). Similarly, using the cosine ratio, we can find the length of MN as 1.9•cos(65).

Therefore, expressions A and B both represent the length of side MN, and they have been obtained by using different trigonometric ratios.

Learn more about expressions

brainly.com/question/14083225

#SPJ11

Hannah has an offer from a credit card issuer for 0% APR for the first 30 days


and 12. 22% APR afterwards, compounded daily. What effective interest rate


is Hannah being offered?

Answers

To find the effective interest rate that Hannah is being offered, we need to take into account the compounding period, which is daily in this case. The effective annual interest rate (EAR) can be calculated using the formula:

EAR = (1 + APR/n)^n - 1

where APR is the annual percentage rate, and n is the number of compounding periods per year.

For the first 30 days, Hannah is offered a 0% APR, so the EAR for this period is simply 0.

After 30 days, Hannah is offered a 12.22% APR compounded daily, which means that there are 365 compounding periods per year. Therefore, the EAR for this period can be calculated as follows:

EAR = (1 + 0.1222/365)^365 - 1

   ≈ 0.1267

So the effective interest rate that Hannah is being offered is approximately 12.67%.

To know more about interest rate , refer here :

https://brainly.com/question/28236069#

#SPJ11

If a point is randomly located on an interval (a, b) and if y denotes the location of the point, then y is assumed to have a uniform distribution over (a, b). a plant efficiency expert randomly selects a location along a 500-foot assembly line from which to observe the work habits of the workers on the line. what is the probability that the point she selects is:closer to the beginning of the line than to the end of the line

Answers

The probability that the point she selects is closer to the beginning of the line than to the end of the line is 0.5 or 50%.



If a point is randomly located on an interval (a, b), and y denotes the location of the point, then y is assumed to have a uniform distribution over (a, b). In this case, the interval is the assembly line of length 500 feet, where a is the beginning and b is the end of the line.

The question asks for the probability that the point she selects is closer to the beginning of the line than to the end of the line. For the point to be closer to the beginning, it must be located in the first half of the line, which is an interval of length 250 feet (500/2).

Since the point has a uniform distribution, the probability of the point being within any sub-interval is equal to the length of the sub-interval divided by the total length of the interval (500 feet).

So, the probability that the point she selects is closer to the beginning of the line than to the end of the line is the length of the first half (250 feet) divided by the total length (500 feet).

Probability = (Length of the first half) / (Total length)
Probability = (250 feet) / (500 feet)
Probability = 0.5 or 50%

There is a 50% chance that the place she chooses will be closer to the line's beginning than its finish.

Learn more about "probability": https://brainly.com/question/13604758

#SPJ11

Water began leaking from a holes in a bucket at a constant rate. Here is a Table of how many ounces were in the bucket at

different times. 10 am:

304 ounces

Noon :

228 ounces

3 pm :

114 ounces


there are more then one question to answer


What is the amount of water that leaks from the bucket each hour?


How many ounces of water were in the bucket at 8 am. ?\


At what time will the bucket be empty. ?


Write the equation of this function in slope intercept form. Use x for the amount of time in hours since the leak began and y for the amount of water in the buck

Answers

The amount of water that leaks from the bucket each hour is 38 ounces/hour.

The amount of water in the bucket at 8 am would have been 380 ounces. The bucket will be empty after 8 hours, or at 6 pm.

To find the amount of water that leaks from the bucket each hour, we can use the information that the leak is at a constant rate. We can find the total amount of water that leaked out of the bucket from 10 am to 3 pm, which is 304 - 114 = 190 ounces.

This is over a time period of 5 hours (from 10 am to 3 pm), so the amount of water that leaks from the bucket each hour is:

190 ounces ÷ 5 hours = 38 ounces/hour

To find how many ounces of water were in the bucket at 8 am, we need to estimate the amount of water that leaked out from 8 am to 10 am. We know that the bucket loses 38 ounces of water every hour, so from 8 am to 10 am (2 hours), the amount of water that leaked out would be:

38 ounces/hour x 2 hours = 76 ounces

Therefore, the amount of water in the bucket at 8 am would have been:

304 ounces + 76 ounces = 380 ounces

To find at what time the bucket will be empty, we can assume that the leak rate remains constant at 38 ounces/hour. We know that the bucket starts with 304 ounces, so we can set up the equation:

y = 304 - 38x

where y is the amount of water in the bucket and x is the time in hours since the leak began. When the bucket is empty, y will be zero, so we can solve for x:

0 = 304 - 38x

38x = 304

x = 8

Therefore, the bucket will be empty after 8 hours, or at 6 pm.

The equation for the amount of water in the bucket as a function of time can be written in slope-intercept form as:

y = -38x + 304

where the slope (m) is -38 (the rate at which water is leaking out of the bucket) and the y-intercept (b) is 304 (the initial amount of water in the bucket).

To know more about equation, refer to the link below:

https://brainly.com/question/29987959#

#SPJ11

A line includes the points (0,-7) and (n, -8) has a slope of -1/6. What is the value of n?

Answers

Answer:

n = 6.

Step-by-step explanation:

The slope of the line = (y2 - y1) / (x2 - x1) where the 2 points are (x1, y1) and (x2, y2).

So, (-8 - (-7)) / (n - 0) = -1/6

-1/n = -1/6

n = 6.

Consider the circle centered at the origin and passing through the point (0, 4)

Answers

Equation of the circle: x^2 + (y - 2)^2 = 4

How to find the equation of the circle?

The circle centered at the origin and passing through the point (0, 4) can be represented by the equation of a circle. The general equation of a circle is (x - h)^2 + (y - k)^2 = r^2, where (h, k) represents the center of the circle and r represents the radius.

Since the center is at the origin (0, 0), the equation simplifies to x^2 + y^2 = r^2. To determine the radius, we can use the point (0, 4) that lies on the circle. Substituting these coordinates into the equation, we get 0^2 + 4^2 = r^2. Simplifying, we find that 16 = r^2.

Therefore, the equation of the circle centered at the origin and passing through the point (0, 4) is x^2 + y^2 = 16.

Learn about more origin

brainly.com/question/24281628

#SPJ11

James says the fraction 3 4 has the same value as the expression 4 ÷ 3. Use the drop-down menus to state whether you agree or not, and why. James is Choose. . A fraction can be interpreted as division of the Choose. By the Choose.

Answers

James says the fraction 3/4 has the same value as the expression 4 ÷ 3. I disagree with James' statement.

The fraction 3/4 is not the same as the expression 4 ÷ 3. A fraction can be interpreted as division of the numerator (top number) by the denominator (bottom number). In this case, 3/4 represents the division of 3 by 4, whereas 4 ÷ 3 represents the division of 4 by 3. These two expressions have different values and are not equal.

Any number of equal parts is represented by a fraction, which also represents a portion of a whole. A fraction, such as one-half, eight-fifths, or three-quarters, indicates how many components of a particular size there are when stated in ordinary English.

More on fractions: https://brainly.com/question/12220774

#SPJ11

Solve each system by substitution
Y=-7x-24
Y=-2x-4

Answers

Answer:

(- 4, 4 )

Step-by-step explanation:

y = - 7x - 24 → (1)

y = - 2x - 4 → (2)

substitute y = - 2x - 4 into (1)

- 2x - 4 = - 7x - 24 ( add 7x to both sides )

5x - 4 = - 24 ( add 4 to both sides )

5x = - 20 ( divide both sides by 5 )

x = - 4

substitute x = - 4 into either of the 2 equations and evaluate for y

substituting into (1)

y = - 7(- 4) - 24 = 28 - 24 = 4

solution is (- 4, 4 )

8. (02.03 mc)


costs of attendance


category


dollar amount


annual tuition and fees


$4,934.00


annual room and board


$1,424.00


annual cost of books and supplies $1,250.00


other one-time fee


$275.00


annual scholarship and grants


$5,250.00


using the information from the table, identify the equation in slope-intercept form that models the total cost of attendance. (1 point)


o y = 2,358x + 275


o y = 2,633x


o y = 7,608x + 275


o y = 7,883

Answers

The equation in slope-intercept form that models the total cost of attendance is: y = 2,633x + 275.


1. Add up the annual costs: tuition and fees ($4,934), room and board ($1,424), and cost of books and supplies ($1,250) to get the total annual cost: $4,934 + $1,424 + $1,250 = $7,608.


2. Subtract the annual scholarship and grants from the total annual cost: $7,608 - $5,250 = $2,358. This is the slope (x) of the equation, as it represents the cost per year.


3. The other one-time fee ($275) is the y-intercept of the equation, as it's a fixed cost that does not change with the number of years.


4. Put the slope and y-intercept into the slope-intercept form (y = mx + b) to get: y = 2,633x + 275.

To know more about slope-intercept form click on below link:

https://brainly.com/question/29146348#

#SPJ11

what are the coefficients in the expression (2x+15)(9x-3) need it asap

Answers

Answer:

2x and 9x are the coefficients.

Step-by-step explanation:

In mathematics, a coefficient is a multiplicative factor in some term of a polynomial, a series, or any expression; it is usually a number, but may be any expression (including variables such as a, b and c ).

Recent studies show that the number of​ three-legged frogs in a particular area is increasing due to exposure to chemical pollutants. The first set of data reported in 2000 estimates a population of 5000​ three-legged frogs. Statistics show an annual increase of​ 15%. Let denote the number of​ three-legged frogs projected to inhabit this area in the year 2000N. How many​ three-legged frogs are projected to inhabit this area by​ 2009? Round to the nearest whole number

Answers

By 2009, it is projected that approximately 13,956 three-legged frogs will inhabit the area.

Recent studies have indicated a growing concern for the population of three-legged frogs in a specific area, as they have been exposed to chemical pollutants. In the year 2000, data estimated that there were about 5,000 three-legged frogs (N) in this area. With an annual increase of 15%, we can project the number of frogs in future years using the formula:

Future population = N * (1 + growth rate) ^ number of years

In this case, we want to determine the number of three-legged frogs in the area by 2009. To calculate this, we will use the given values:

Future population = 5,000 * (1 + 0.15) ^ (2009 - 2000)

Future population = 5,000 * (1.15)⁹

Future population ≈ 13,956

Therefore, by 2009, it is projected that approximately 13,956 three-legged frogs will inhabit the area, rounding to the nearest whole number. This increase in population highlights the potential ecological consequences of chemical pollutants on the environment and the need for further investigation and mitigation measures.

To know more about projection, refer to the link below:

https://brainly.com/question/15583437#

#SPJ11

A dth tv connection provides channels in english and other languages in the ratio 7:13. what percentage of the channels are in english

Answers

A DTH TV connection provides channels in English and other languages in the ratio 7:13. To find out what percentage of the channels are in English, you need to divide the number of English channels by the total number of channels and then multiply the result by 100.

Let's assume that there are a total of 100 channels available on this DTH TV connection. According to the given ratio, 7 out of every 20 channels will be in English. So, the percentage of channels in English will be:

(7/20) x 100 = 35%

Therefore, 35% of the channels on this DTH TV connection are in English.

To know more about TV connection refer here

https://brainly.com/question/30629673#

#SPJ11

What is the median of the lower half of data

Answers

We can see here that in order to find the median of the lower half of data, one will have to sort out the data in an ascending order. Then take the  lower half of the data (i.e., the first half of the sorted data) and find its median.

What is median?

The median, which is used to measure central tendency in statistics, is the point at which a dataset may be divided into two equal parts. If a dataset has an even number of values, it is the average of the two middle values or the middle value in a sorted dataset.

The values in the dataset must first be arranged from lowest to highest in order to determine the median. The median is the middle value if the dataset has an odd number of values.

Learn more about median on https://brainly.com/question/26177250

#SPJ1

3. The scale of a room in a blueprint is 2 inches : 1 foot. A window in the same blueprint is 12 inches. Complete the table. Blueprint Length (in.) Actual Length (ft) a. How long is the actual window? 2 1 4 3 4 10 12 5 6 b. A mantel in the room has an actual width of 8 feet. What is the width of the mantel in the blueprint?​

Answers

Therefor, the length of mantel in blueprint is > 30 ft

width of the mantel in the blueprint 8ft×2inc/1ft=16inch

what is width?

The term "width" refers to the length from side to side of anything. For instance, the shorter side of a rectangle would be the width.

we know that

[scale]=[blueprint]/[actual]-------> [actual]=[blueprint]/[scale]

[scale]=3/5 in/ft

for [wall blueprint]=18 in

[wall actual]=[wall blueprint]/[scale]-------> 18/(3/5)----> 30 ft

Part A)

the actual wall is 30 ft  long

Part B) window has actual width of 2.5 ft

[ window blueprint]=[scale]*[actual window]-----> (3/5)*2.5----> 1.5 in

the width of the window in the blueprint is 1.5 in

Part C) Complete the table

For [blueprint length]=4 in

[actual length]=[blueprint length]/[scale]-------> 4/(3/5)----> 20/3 ft

For [blueprint length]=5 in

[actual length]=[blueprint length]/[scale]-------> 5/(3/5)----> 25/3 ft

For [blueprint length]=6 in

[actual length]=[blueprint length]/[scale]-------> 6/(3/5)----> 30/3=10 ft

For [blueprint length]=7 in

[actual length]=[blueprint length]/[scale]-------> 7/(3/5)----> 35/3 ft

For [actual length]=6 ft

[blueprint length]=[actual length]*[scale]-------> 6*(3/5)----> 18/5 in

For [actual length]=7 ft

[blueprint length]=[actual length]*[scale]-------> 7*(3/5)----> 21/5 in

For [actual length]=8 ft

[blueprint length]=[actual length]*[scale]-------> 8*(3/5)----> 24/5 in

For [actual length]=9 ft

[blueprint length]=[actual length]*[scale]-------> 9*(3/5)----> 27/5 in

B) width of the mantel in the blueprint 8ft×2inc/1ft=16inch

To know more about width visit:

brainly.com/question/30173060

#SPJ1

Consider the function f(x) = 2x³ + 6x² – 144x + 4, -6 ≤ x ≤ 5. Find the absolute minimum value of this function. Answer: Find the absolute maximum value of this function. Answer:

Answers

The absolute maximum value of the function f(x) is 222.

To find the absolute minimum value of the function f(x), we need to first find the critical points within the given interval -6 ≤ x ≤ 5. To do this, we take the derivative of f(x) and set it equal to zero:

f'(x) = 6x² + 12x - 144
0 = 6(x² + 2x - 24)
0 = 6(x+6)(x-4)

The critical points are x=-6, x=-4, and x=4. To determine which of these points correspond to a minimum value, we evaluate f(x) at each of these points and at the endpoints of the interval:

f(-6) = -880, f(-4) = -184, f(4) = -136, f(-6) = -880, f(5) = 222

Therefore, the absolute minimum value of the function f(x) is -880.

To find the absolute maximum value of the function f(x), we follow the same process. The critical points are still x=-6, x=-4, and x=4, but now we need to evaluate f(x) at each of these points and at the endpoints of the interval to determine which corresponds to a maximum value:

f(-6) = -880, f(-4) = -184, f(4) = -136, f(-6) = -880, f(5) = 222

Therefore, the absolute maximum value of the function f(x) is 222.

To learn more about critical points, refer below:

https://brainly.com/question/7805334

#SPJ11

help i need this done pls 50 points

Answers

The length of the diagonal is 12. 7in

How to determine the length

To determine the length of the diagonal, we need to know the Pythagorean theorem.

The Pythagorean theorem states that the square of the hypotenuse side is equal to the sum of the squares of the other two sides of a triangle.

The other two sides are the opposite and the adjacent sides.

From the information given in the diagram, we have that;

The opposite side = 12in

The adjacent side = 4in

Substitute the values

x² = 12² + 4²

find the squares

x² = 160

find the square root

x = 12. 7 in

Learn about Pythagorean theorem at: https://brainly.com/question/654982

#SPJ1

What is -2(x + 12y - 5 - 17x - 16y + 4) simplified?

-40x + 8y + 2

28x + 8y +2

28x + 6y + 2

-28x - 8y + 2

Answers

Answer:

Step-by-step explanation:

First, we can simplify the expression inside the parentheses by combining like terms:

-2(x + 12y - 5 - 17x - 16y + 4) = -2(-16x - 4y - 1)

Next, we can distribute the -2 to each term inside the parentheses:

-2(-16x - 4y - 1) = 32x + 8y + 2

Therefore, -2(x + 12y - 5 - 17x - 16y + 4) simplified is 32x + 8y + 2.

The simplified expression is 32x + 8y + 2.

Simplification of an algebrai expression can be defined as the process of writing an expression in the most efficient and compact form without affecting the value of the original expression.

The process entails collecting like terms, which implies adding or subtracting terms in an expression.

Simplify the expression -2(x + 12y - 5 - 17x - 16y + 4).

First, let's distribute the -2 to each term inside the parentheses:

-2(x) + (-2)(12y) - (-2)(5) - (-2)(17x) - (-2)(16y) + (-2)(4)

Now we'll multiply: -2x - 24y + 10 + 34x + 32y - 8

Next, we'll combine like terms:

(-2x + 34x) + (-24y + 32y) + (10 - 8)

The result is 32x + 8y + 2

Learn more about expression in math at

https://brainly.com/question/30091997

#SPJ11

expand and simplify(2w-3)3

Answers

Answer:

6w-9

Step-by-step explanation:

2w×3=6w

-3×3=-9

=6w-9

Resume the totat revenue from the sale of them is given by R(x) * 25 1n (6x + 1), while the total cost to produce x items is C(x)=ſ. Find the approximate number of items that should be manufactured so that profit, RIX-C) is maximum G A 143 Rems OB. 84 items C. 47 items OD 114 items

Answers

The approximate number of items that should be manufactured so that the profit, P(x) = R(x) - C(x), is maximum is 47 items (option C).

To find the approximate number of items that should be manufactured to maximize profit, we need to first find the profit function P(x) by subtracting the total cost, C(x), from the total revenue, R(x). Then, we need to find the critical points of P(x) and determine which one corresponds to the maximum profit.

process of finding profit:

Step 1: Find the profit function P(x) = R(x) - C(x)
Given R(x) = 25 ln(6x + 1) and C(x) = ∫x, let's find P(x):
P(x) = R(x) - C(x)
P(x) = 25 ln(6x + 1) - ∫x
Step 2: Find the critical points of P(x)
To find the critical points, we need to take the derivative of P(x) and set it equal to 0:
P'(x) = d/dx [25 ln(6x + 1) - ∫x]
Since the derivative of ln(6x + 1) is (6/(6x + 1)), and the derivative of ∫x is x:
P'(x) = 25 [tex]\times[/tex] (6/(6x + 1)) - x
Now, set P'(x) = 0 and solve for x:
25 [tex]\times[/tex] (6/(6x + 1)) - x = 0
Step 3: Determine which critical point corresponds to the maximum profit
The approximate number of items that should be manufactured so that the profit, P(x) = R(x) - C(x), is maximum is 47 items (option C).

To know more about Profit:

https://brainly.com/question/15036999

#SPJ11

find the critical numbers of f(x)=4−5x/4 + x and classify any local extrema.

Answers

The function has a global maximum at the vertex (2/5, 41/25), and there are no local maxima or minima.

To find the critical numbers of the function f(x) = 4 - 5x/4 + x, we first need to find its derivative:

f'(x) = -5/4 + 1

f'(x) = -1/4

To find the critical numbers, we set f'(x) equal to zero and solve for x:

-1/4 = 0

This is never true, so there are no critical numbers for f(x).

Since there are no critical numbers, there are no local maxima or minima for the function. Instead, we can analyze the behavior of the function to determine if it has any extrema.

One way to do this is to examine the end behavior of the function. As x approaches positive or negative infinity, the leading term of the function is -5x/4, which dominates the constant term. Therefore, as x becomes large in either direction, the function approaches negative infinity. This suggests that the function has a global maximum at its vertex.

To find the vertex, we can complete the square:

f(x) = 4 - 5x/4 + x

[tex]f(x) = -(5/4)x^2 + x + 4[/tex]

[tex]f(x) = -(5/4)(x^2 - (4/5)x) + 4[/tex]

[tex]f(x) = -(5/4)(x - 2/5)^2 + 4 + (5/4)(2/5)^2\\f(x) = -(5/4)(x - 2/5)^2 + 41/25[/tex]

Therefore, the function has a global maximum at the vertex (2/5, 41/25), and there are no local maxima or minima.

For more such questions on global  maximum :

https://brainly.com/question/29258664

#SPJ11

In an effort to eat healthier, Bridget is tracking her food intake by using an application on her phone. She records what she eats, and then the


application indicates how many calories she has consumed. One Monday, Bridget eats 10 medium strawberries and 8 vanilla wafer cookies as an


after-school snack. The caloric intake from these items is 192 calories. The next day, she eats 20 medium strawberries and 1 vanilla wafer cookie as an after-school snack. The caloric intake from these items is 99 calories.



a. Write a system of equations for this problem situation. Let S represent the number of calories in one strawberry and let W represent the number of calories in one vanilla wafer cookie.



The equation _____ represents the calories Bridget ate on Monday and the equation _____ represents the calories she ate the next day.



b. Solve the system of equations using the substitution method. Check your work.



The number of calories in each strawberry is ____


And the number of calories in each vanilla wafer cookie is ____. The solution is ____.



PLEASE HELP ME

Answers

The equation 10S + 8W = 192 represents the calories Bridget ate on Monday and the equation 20S + 1W = 99 represents the calories she ate the next day.

The number of calories in each strawberry is 4, and the number of calories in each vanilla wafer cookie is 19.

a. We have two equations for the two days, using S for the number of calories in a strawberry and W for the number of calories in a vanilla wafer cookie:

On Monday:
10S + 8W = 192

On Tuesday:
20S + 1W = 99

b. To solve the system of equations using the substitution method, first solve one of the equations for one of the variables. We'll choose the second equation and solve for W:

W = 99 - 20S

Now substitute this expression for W in the first equation:

10S + 8(99 - 20S) = 192

Expand and simplify:

10S + 792 - 160S = 192

Combine like terms:

-150S = -600

Now divide by -150:

S = 4

Now that we have the value for S, substitute it back into the expression for W:

W = 99 - 20(4)

W = 99 - 80

W = 19

So the number of calories in each strawberry is 4, and the number of calories in each vanilla wafer cookie is 19. The solution is (S, W) = (4, 19).

To learn more about expression, refer below:

https://brainly.com/question/14083225

#SPJ11

The box plot displays the number of push-ups completed by 9 students in a PE class.

A box plot uses a number line from 12 to 58 with tick marks every 2 units. The box extends from 27.5 to 42.5 on the number line. A line in the box is at 37. The lines outside the box end at 15 and 55. The graph is titled Push-Ups In PE and the line is labeled Number of Push-Ups.

Which of the following represents the value of the lower quartile of the data?

27.5
37
42.5
55

Answers

Given that 27.5 is the bottom end of the box, the lower quartile's value equation (Q1) must fall between 27.5 and 30. Hence, the appropriate response is 37.

Since, A mathematical equation links two statements and utilizes the equals sign (=) to indicate equality.

In algebra, an equation is a mathematical assertion that proves the equality of two mathematical expressions.

For instance, in the equation 3x + 5 = 14, the equal sign separates the numbers by a gap. A mathematical formula may be used to determine how the two sentences on either side of a letter relate to one another. The logo and the particular piece of software are usually identical. like, for instance, 2x - 4 = 2.

Here, 25% of the data fall inside the lower quartile (Q1), which is represented by that number. Q1 is situated near the bottom of the box in the box plot.

According to the description, the box's boundary is at 30, and its size ranges from 27.5 to 42.5 on the number line. As a result, the median value of the middle 50% of the data is 37, with a range of 27.5 to 42.5.

There are some data points outside the middle 50% since the lines outside the box finish at 15 and 55.

Hence; 27.5 is the bottom end of the box, the lower quartile's value (Q1) must fall between 27.5 and 42.5.

Hence, the appropriate response is,

= 37

To know more about equation visit:

brainly.com/question/649785

#SPJ1

Find f'(4) for f(x) 8/In(3x^2) Round to 3 decimal places, if necessary.

Answers

To find f'(4), we need to take the derivative of f(x) with respect to x and then evaluate it at x=4. Using the chain rule, we get:
f'(x) = -16x/(ln(3x^2))^2

So, f'(4) = -16(4)/(ln(3(4)^2))^2 = -64/(ln(48))^2

Rounding to 3 decimal places, we get f'(4) = -0.019.
To find f'(4) for f(x) = 8/ln(3x^2), we first need to differentiate f(x) with respect to x. We will use the quotient rule and the chain rule for this purpose.

The quotient rule states: (u/v)' = (u'v - uv')/v^2, where u = 8 and v = ln(3x^2).

Now, differentiate u and v with respect to x:
u' = 0 (since 8 is a constant)
v' = d(ln(3x^2))/dx = (1/(3x^2)) * d(3x^2)/dx (using chain rule)

Now, differentiate 3x^2 with respect to x:
d(3x^2)/dx = 6x

So, v' = (1/(3x^2)) * (6x) = 2/x

Now, apply the quotient rule for f'(x):
f'(x) = (0 - 8 * (2/x))/(ln(3x^2))^2 = -16/(x * (ln(3x^2))^2)

Now, plug in x = 4 to find f'(4):
f'(4) = -16/(4 * (ln(3*(4^2)))^2) = -16/(4 * (ln(48))^2)

Rounded to 3 decimal places, f'(4) ≈ -0.171.

Visit here to learn more about derivative  : https://brainly.com/question/25324584
#SPJ11

Determine the equation of the tangent plane to the surface z = y ln x at the point (1, 4, 0)

Answers

z = 4(x - 1) - ln (y - 4)

Therefore, the equation of the tangent plane to the surface z = y ln x at the point (1, 4, 0) is z = 4(x - 1) - ln (y - 4).

To determine the equation of the tangent plane to the surface z = y ln x at the point (1, 4, 0), we first need to find the partial derivatives of the surface with respect to x and y.

∂z/∂x = y/x

∂z/∂y = ln x

Then, we can use these partial derivatives along with the point (1, 4, 0) to find the equation of the tangent plane using the formula:

z - z0 = ∂z/∂x(x0, y0)(x - x0) + ∂z/∂y(x0, y0)(y - y0)

where (x0, y0, z0) is the given point.

Plugging in the values, we have:

z - 0 = (4/1)(x - 1) + ln 1(y - 4)

Simplifying:

z = 4(x - 1) - ln (y - 4)

Therefore, the equation of the tangent plane to the surface z = y ln x at the point (1, 4, 0) is z = 4(x - 1) - ln (y - 4).

Learn more about tangent plane here:

https://brainly.com/question/30260323

#SPJ11

A force of 80 pounds on a rope is used to pull a box up a ramp inclined at 10 degrees from the horizontal. The rope forms an angle of 33 degrees with the horizontal. How much work is done pulling the box 26 feet along the ramp?

Answers

The work done on the displacement is 301.95J

What is the work done in pulling the box

To determine the work done, we need to find the displacement in which the box moved.

cos θ = adjacent / hypothenuse

cos 33 = adjacent / 80

adjacent = 80 * cos 33

adjacent = 67.1 lbs

The force applied is 67.1lbs

The displacement on the ramp;

sin θ = opposite / hypothenuse

sin 10 = opposite / 26

opposite = 26 * sin 10

opposite = 4.5 ft

The work done in moving the object can be calculated as;

work done = force * displacement

work done = 67.1 * 4.5

work done = 301.95 J

Learn more on work done here;

https://brainly.com/question/25573309

#SPJ1

How many 4-digit numbers have the second digit even and the fourth digit at least twice the second digit?

Answers

There are 1350 4-digit numbers that have the second digit even and the fourth digit at least twice the second digit.

To form a 4-digit number, we have 10 choices for each digit, except the first digit, which can't be 0. Hence, there are 9 choices for the first digit.

For the second digit, there are 5 even digits (0, 2, 4, 6, 8) to choose from.

For the third digit, there are 10 choices.

For the fourth digit, we can choose any of the even digits we picked for the second digit, or any of the larger odd digits 4, 6, 8.

Hence, the number of 4-digit numbers that meet the given criteria is

9 × 5 × 10 × 3 = 1350.

Therefore, there are 1,350 4-digit numbers that have the second digit even and the fourth digit at least twice the second digit.

To know more about 4-digit numbers:

https://brainly.com/question/679725

#SPJ4

In the diagram shown, segments AE and CF are both perpendicular to DB.
DE=FB, AE=CF. Prove that ABCD is a parallelogram.

Answers

Answer:

Step-by-step explanation:

Given:- ABCD is a parallelogram, and AE and CF bisect ∠A and ∠C respectively. To prove:- AE∥CF Proof:- Since in a parallelogram, opposite angles are equal.

Tell whether the angles are adjacent or vertical. Then find the value of x. Please help with this question

Answers

Answers - adjacent angles, x = 63 degrees

Explanation

Adjacent angles share a side, vertical angles are across from each other so this is adjacent

This is a straight angle that includes these adjacent angles, a straight angle sum is 180 degrees.

180 - 117 = 63

X = 63 degrees
Answers - adjacent angles, × = 63 degrees

Find the error & explain why it is wrong:
megan solved the following problem. what did she do wrong?
what is (f - g)(2)?
f(x) = 3x2 – 2x + 4
g(x) = x2 – 5x + 2

Answers

The value of (f-g)(2) is 16, provided that Megan has made no mistakes in the calculation.

Find the error in the given problem solved by Megan?

The problem asks us to compute the value of (f - g)(2) where f(x) = 3x^2 - 2x + 4 and g(x) = x^2 - 5x + 2.

The notation (f - g)(2) means that we need to subtract g(x) from f(x) and then evaluate the result at x = 2. We can do this as follows:

(f - g)(x) = f(x) - g(x) = (3x^2 - 2x + 4) - (x^2 - 5x + 2) = 2x^2 + 3x + 2

Substituting x = 2, we get:

(f - g)(2) = 2(2)^2 + 3(2) + 2 = 16

Therefore, the value of (f - g)(2) is 16.

It's worth noting that the problem statement mentions "what did she do wrong?" without providing any context or information about what Megan did or didn't do. So, it's not possible to identify any error in Megan's solution based on the given information. However, based on the correct computation above, we can be sure that (f - g)(2) is indeed equal to 16.

In other words, it can be described as,

The error in Megan's solution is not clear from the given statement. However, it seems that she may have made an error while computing (f-g)(2).

To compute (f-g)(2), we need to subtract g(2) from f(2) as follows:

f(2) = 3(2)^2 - 2(2) + 4 = 12

g(2) = (2)^2 - 5(2) + 2 = -4

Therefore, (f-g)(2) = f(2) - g(2) = 12 - (-4) = 16. is the final conclusion.

Learn more about Solution

brainly.com/question/30198131

#SPJ11

Other Questions
Jill has $1275.00 in her savings account. When she opened her account, she had $300.Every week she deposited $75.00. How many weeks did it take to earn $1275.00? Co-culture members who take an ______ approach most intensely defend their beliefs and traditions. a. applied b. aggressive c. active d. assertive Can you help me with this question step by step. Which is a possible result of higher air temperatures caused by global warming?increase in sea levelO freezing of polar ice capsO rapid evaporation of water in the soilO flooding of low-lying coastal areas darrell owns a furniture store. his total costs are $225,000 per year, and his fixed costs are $150,000 per year. this means that his variable costs are: if y=x^2+2x-5 for -3 COMPARE BY USING ,=,= Which rhetorical appeal is used in the following sentence from paragraph 2 of the passage?"A study found that the drinking water in schools in 27 states had lead and other chemicals above the safe level."ResponsesA pathospathosB logoslogosC kairoskairosD ethosethos Two ice skaters, starting from rest, hold onto the ends of a 10 m pole. A 40-kg player is at one end of the pole and a 60-kg player is at the other end. The players then start pulling themselves along the pole towards each other while sliding without friction on the ice. If the two skaters continue past each other after they meet, what distance will the 60-kg player have moved with respect to the ice when the skaters have exchanged positions with respect to each other?. Lab Report: Titration HELP!!! I dont understand this!! Anyone done this before?? C CA student believes that a certain number cube is unfair and is more likely to land with a six facing up. The student rollsthe number cube 45 times and the cube lands with a six facing up 12 times. Assuming the conditions for inferencehave been met, what is the 99% confidence interval for the true proportion of times the number cube would land with asix facing up?0. 27 2. 580. 221-0. 27)450. 7342. 330. 731-0. 73)450. 27 2. 330. 271 -0. 20)450. 73 +2. 580. 73(10. 73)45MixSave and Exit Solve the following inequality for r. Write your answer in simplest form. -5r - 2(-10r - 9) A 16-bit periodic count-down timer uses a clock source of 2khz and clock divider of 2, choose proper options forhow much is the frequency of the clock that feeds the counter inside this timer? [ select ] ["1 khz", "1 ms", "2 khz", "0.5 ms"]what is the largest load value for this timer? [ select ] ["2^16 - 1", "2^16", "2^16 + 1"]based on the answer to part 2, approximately, how long is the longest period for this periodic timer? [ select ] ["65.536 s", "0.5 ms", "1 ms", "(2^16) s"]assume the load value is set at 999 and no rollover has happened between events, e1 and e2. if the counter reading (the value inside the counter) for the two events, c1 and c2, are 550 and 200, how long has elapsed between the two events? [ select ] ["350 ms", "350 sys clock cycles"]assume the load value is 9999. once an event, e1, happens, the light should turn on and stay on for 3 seconds. if the counter value when e1 happens is 2000 and we immediately turn on the light, what should be the counter value when we have to turn off the light (after 3 seconds)? TEXT:Economist 18991992Friedrich August von Hayek was an Austrian-born British economist and writer whose career spanned 70 years. Born into a wealthy Austrian family, Hayek attended the University of Vienna, where he studied law, psychology, and economics. He taught at the London School of Economics as well as the University of Chicago. In 1974, Hayek received the Nobel Prize for Economics.Hayek is well known for his sometimes-controversial criticisms of socialism and Keynesian welfare policies. His most famous book, The Road to Serfdom, was published in 1944. In it, he argued that attempts at economic planning inadvertently impose values on society. He felt that such plans could not exist unless economic planners exercised political control.One of the leading members of the Austrian School of Economics, Hayek endorsed free market economics and opposed widespread government intervention in a time when many intellectuals favored socialism and Keynesian economics. However, after the stagflation crisis of the 1970s made many question the validity of Keynesian economic theory, Hayek's ideas experienced a major growth in popularity.Critical Thinking QuestionsHypothesizing Write a paragraph discussing whether you think Hayek would approve of a government-sponsored economic stimulus. Give reasons for your view.Analyzing Write a short essay explaining how Hayek's view of the importance of a free price system fits in with his laissez-faire philosophy. which biome is known for being hot and moist with constant rain The Indian Super League is Indias leading soccer league but is struggling withviewership? What could be the reasons for this lack of consumer interest? Whatstrategies would you recommend to the league management if you were its strategyconsultant? A negatively charged balloon moves close to another balloon. They then repel each other. What can be said about the other balloon? (2 points)A: Both balloons have a positive charge. B: It has a negative charge. C: The balloon is uncharged. D: There is a positive charge. An object vibrates with a frequency of 2200 Hz to produce sound waves that travel through air with a speed of 340 m/s. Adjacent compressions (i.e., wavelengths) in the sound wave are a distance of meters apart. How does the setting of the story enhance the overall mood? Cite evidence from the text to support your answer.article, "Beware the kitten holy." Find the value of X of the circle